Inscription / Connexion Nouveau Sujet
Niveau Maths sup
Partager :

somme

Posté par
xunil
15-11-08 à 17:20

bonsoir,

S=\bigsum_{k=0}^nk^3+k^2

bon je sépare ma somme je tombe sur deux sommes usuelles ok !

mais si je veux faire comme ca:

Soit S_x=\bigsum_{k=0}^nk^3x^{k}+\bigsum_{k=0}^nk^2x^k

par exemple, \bigsum_{k=0}^nk^2x^k=x^2(\bigsum_{k=0}^nk(k-1)x^{k-2}+\bigsum_{k=0}^nkx^{k-2})

= x^2(\bigsum_{k=0}^nk(k-1)x^{k-2})+x\bigsum_{k=0}^nkx^{k-1})

=x^2(\bigsum_{k=0}^nkx^{k})''+ x(\bigsum_{k=0}^nkx^{k})'

(\bigsum_{k=0}^nkx^{k})=\frac{x^n-1}{x-1} mais cela vraie pour x\neq 1 ... ce qui m'embête.

donc juste pour savoir si je peux m'en sortir avec ma seconde méthode.

merci

Posté par
Tigweg Correcteur
re : somme 15-11-08 à 17:51

Bonjour,

oui c'est possible, hormis le fait que lorsque tu factorises x² dans tes deux sommes, les indices k ne varient plus qu'entre 2 et n, sinon ça n'a pas de sens.

Pour conclure, utilise que les fonctions écrites dans chaque membre sont égales pour x différent de 1, et que celle de gauche est continue en 1.
Sa valeur en 1 est donc égale à la limite lorsque x tend vers 1 du membre de droite, qui peut par exemple s'obtenir en reconnaissant la limite du taux de variation en x = 1 d'une fonction dérivable en 1.

Posté par
xunil
salutations Tigweg 15-11-08 à 17:58

oué mince gaffe benh je reregarde et reviens au cas où...

merci

Posté par
xunil
re : somme 15-11-08 à 19:02

en fait ca m'embete.

je sais que ma fonction en 1 fait n mais à la rigueur...

en fait je dois dériver mon expression \frac{x^n-1}{x-1} deux fois et c'est là que je passe à la limite en 1 ?

je te dis cela de suite.

Posté par
Tigweg Correcteur
re : somme 16-11-08 à 07:29

Oui, puis raisonnement analogue à ce que j'ai écrit pour trouver la limite en 1 de l'expression dérivée deux fois.
Par contre, je ne m'en étais pas aperçu hier en répondant rapidement à ton post, mais l'expression de départ n'est pas égale à 3$\rm x^2(\bigsum_{k=2}^nkx^{k})''+ x(\bigsum_{k=2}^nkx^{k})' .

Je pense que tu as plutôt voulu dire 3$\rm x^2(\bigsum_{k=2}^n x^{k})''+ x(\bigsum_{k=2}^n x^{k})' .

Posté par
Tigweg Correcteur
re : somme 16-11-08 à 07:33

Pardon, la deuxième somme démarre à k = 1, seule la première démarre à k = 2 !

Posté par
xunil
re : somme 16-11-08 à 15:39

c'est bon ! je m'en suis sorti.

merci Tigweg

@+

Posté par
Tigweg Correcteur
re : somme 16-11-08 à 20:40

Parfait, avec plaisir xunil!



Vous devez être membre accéder à ce service...

Pas encore inscrit ?

1 compte par personne, multi-compte interdit !

Ou identifiez-vous :


Rester sur la page

Inscription gratuite

Fiches en rapport

parmi 1677 fiches de maths

Désolé, votre version d'Internet Explorer est plus que périmée ! Merci de le mettre à jour ou de télécharger Firefox ou Google Chrome pour utiliser le site. Votre ordinateur vous remerciera !